Monday, July 11, 2022

e-EQE2022: Main Exam - what to do after a pass or a fail

15-07-2022: added comments about translation errors.
14-07-22 - added note about split exams compensation.
13-07-22 - added "fit to practice" check during marking.

A big congratulations to all who passed at least at one exam. None of the exams are easy, and if you are unlucky, it is easy to miss something crucial or to misread a sentence.

The result totals and statistics are available here. The pass rate for B is higher than last year, and D is relatively high compared to previous years (not including D 2021 because of the global compensation for technical issues). 

Unfortunately, (since 2021) anonymized individual results are no longer made available, so we cannot calculate statistics or see the mark distributions per exam. Joeri Beetz has a nice website with extensive statistics for the older exams. We were told last year that this omission was due to a lack of capacity.

2021 "Pass rates" - candidates with 45 points or higher:

  • Nr of candidates: Main Exam = 2780, A = 1905, B = 2005, C = 1852, D = 2003
  • A  = 79%, B = 65%, C = 57%, D = 92% 
  • Passed last exam in 2021: 1093 (= 53 % of all ME candidates)

2022 "Pass rates" - candidates with 45 points or higher:

  • Nr of candidates: Main Exam = 1918, A = 886, B = 1158, C = 1400, D = 704
  • A  = 73%, B = 84%, C = 61%, D = 71% 
  • Passed last exam in 2022: 885 (= 46 % of all ME candidates)

The Examiners Reports for each exam are also available here. Unfortunately, the marking sheets are missing (they were included in 2021 and previous years). These are the same as each candidate receives, with a rough division of  available marks. I don't know why they are not included this year.

For each exam, the Examination Committees have selected Candidates' Answers in English, French and German who scored quite highly with an exam answer close to their desired solution. These are unedited, so they can contain mistakes. Unfortunately, the marking sheets for these candidates are still missing (they were also missing in 2021). These marking sheets were very useful for candidates trying to understand where they missed marks. We were told last year that this omission was due to a lack of capacity. 

Still to be published is the Survey report, summarising the answers provided immediately after the exam. It will appear here, at the bottom of the page. Hopefully, it will be back to it's pre-2020 extensive format. In 2021, it was merely a PR folder - we were told last year that this was due to a lack of capacity. 

In October (usually in week 42), there will be a meeting of Tutor's and the Examination Committees to directly discuss the exams of 2022, and to look ahead to EQE 2023. No definite date for the meeting is yet available. In 2021, it was online - no word yet if it will be back to an in-person gathering.

If you passed your last exam, double congratulations 🥇and welcome to the post-EQE world 💪. 

  • For attorneys, request entry onto the list of representatives. 
  • If you are a national of a non-EPC state and you wish to be on the list, you need to request an exemption under Art. 134(7). This involves extra administration - see the FAQ's. Preston Richard has made a template letter that you can use.
  • Relax and look forward to developing other aspects of your career.
  • Even though the epi has no formal requirement for permanent education, commit to keeping up your knowledge. You still need up-to-date knowledge to provide optimal services to your clients, especially of changes in EPO procedure:
    • Subscribe to a blog that regularly discusses case law and legal changes, like the IP.appify blog
    • Go through the yearly changes to the EPO guidelines and follow G decisions
    • Use your legal knowledge more in your daily work (see if a colleague needs help with an office action or opposition, give a presentation to your office, check whether your office procedures meet "all due care", teach subjects to trainees, write a legal article).
    • Attend or follow the EPO's education sessions related to Applying for a Patent, to Law and Practice, or for Patent Attorneys
    • Become a tutor, or volunteer for an Examination Committee
    • If you have added digital notes to EPC.app and PCT.app, they will be updated automatically each year.
Click on READ MORE > > for possible actions after a fail  
If you failed an exam, 
think carefully about your options 🤔.

  • Realise you are not alone - most candidates take at least 2 years to pass everything. 
    • I passed A, C and D first time (passing D mainly on D2). My very complicated B solution got few marks. The next year, my simple B solution easily passed 😃. I always struggled with "B" - I only passed the equivalent NL attorney national paper at the 3rd attempt 😮. 
  • Don't be embarrassed by failure. Even well-prepared and "brilliant" candidates can fail.
  • You have to decide for yourself what to do. There are no magic formulas that work for everybody, especially when you are balancing family, relationships and work with your study. Look for the path that will make you the happiest. Take some time to think it through. Examples of decisions that candidates have made:
    • if you are unhappy at your current work, but only partially qualified, you can still get another job. In particular, if you have already passed D, a lot of firms will take a chance on you.
    • concentrating on national patent attorney qualifications first instead of EQE, or concentrating on EQE first before national qualifications
    • leaving an attorney firm to join the EPO, or leaving the EPO to join an attorney firm
    • leaving an attorney firm to join a company, or leaving a company to join an attorney firm
    • taking a year off from studying, or immediately registering for all failed exams
    • concentrating on well-being alongside studying for the exam. E.g. running more, eating healthier and, especially, MEDITATING
  • Be aware of what your options are, particularly with compensable fails. The EQE helpdesk will be happy to answer any questions about this.
  • 44 or less is a fail, 45 to 49 is a compensable fail. You can retake any paper that you have failed (49 or less). 
  • Each time that you do a paper, they will check to see the status of all 4 paper scores, and whether you satisfy the EQE passing criteria at that moment.
  • To pass, you need at least 200 marks in total for all exams, with no more than two compensable fails. For example: A: 60, B: 50, C: 45, D: 45 and A: 55, B: 55, C: 45, D: 45 are just enough to pass.
  • Study your exam in detail to see where you went wrong. Compare it to the model solution in the Examiners Report. Why did you not get the other 55 marks that were available? Be honest with yourself about whether your preparation was good enough. Did you focus during the exam on completing the parts with the most marks, or were you doing the parts that you like instead? Were you trying to provide "perfect answers?"
  • Also look at the Candidate's Answer in your preferred language - what did they get that you missed? But be aware that this is usually a native speaker who write a lot (padding and generic statements don't score any marks), managed to get on the wavelength of the Examination Committee, and scored around 80 marks. You do not need this level to pass.
Option 1:  "bank" your compensable fails
  • For a compensable fail, you can keep it in your results "bank" and try to compensate it with a pass of at least 55 in another paper. If you retake a failed paper, the score in your "bank" is always overwritten by the new one, even if it is lower.
  • If you have 45 - 49 for D in your "bank", think carefully before retaking it. It is a lot of work to prepare for D, and you can score above 60 marks in A and B if you are very well prepared. It used to be the same for C, but the current online format and imperfect split makes scoring on C unpredictable. 
Option 2: prepare your study plan to resit
  • If you were well-prepared and you failed, then it is quite a shock. Especially when you see other fellow candidates all pass. Don't be embarrassed by failure. Even well-prepared and "brilliant" candidates can fail.
  • If you were not well-prepared for the first time, then you already know what you need to differently - prepare well.
  • The biggest problem resitters face is confidence. During each paper, you need to take a thousand micro-decisions about where to start, what to write down, what to do now, what to do later, when to stop writing, parts to skip etc. The first time your confidence is sky-high, so you are not afraid to take these decisions. But when you resit, you will be more hesitant to take decisions in case you are "wrong". This can be hard to break, especially if you are not used to a fear of failing.
  • Be careful when resitting D. You need to be able to motivate yourself to go through everything again. A lot of D resitters end up in a vicious cycle of postponing studying until quite late, not being well-prepared and failing again. 
  • In general, D resitters get better on the D2 part, and get worse on the D1 part. This is due to daily work experience advising clients helping you to get through D2 faster, whereas D1 involves being able to find minute details from the law in article/rules that you never encounter in practice. So, if you expect D1 to get worse, make sure you get practice D2 more.
  • You might run out of old papers to practice because you have done them all. But you can learn a lot by redoing them and trying to score more marks in different areas of the paper. You can also go back and do older ones, but the exam formats and the law has changed over the years.
  • If you are doing the exam in a non-native language, you can practice reading, listening and writing in En, Fr or Ger using the Terminology Manual. This is great resource for training patent attorneys, and the examples used are old EQE exams.
Option 3: file an appeal
  • In general, I advise candidates not to do this unless they think that following the long and frustrating process will help them mentally. You may not get a decision until after registration for the next EQE has closed, so you will need to pre-emptively register. In most Main Exam cases, you will only get a final decision after the next EQE has been held. So, you should not do it if it will affect your preparation for the next EQE.
  • The details of the appeal procedure are not publicly documented, so this is what I know from thee handful of appeals that I have seen over the years. If you are in doubt about any of these points, check with the EQE helpdesk. 
  • The exam preparation and marking is pretty thorough, so the chances of winning an appeal by means of the Disciplinary Board of Appeal are quite small. But the procedure does guarantee that your appeal letter is read and considered by the Examination Board. 
  • There is one fee that you need to pay to challenge the decision, and that covers challenges of the marks awarded to 1 - 4 of your papers. It is set at 6x the examination fee - R. 9, IPREE. You will get your fee back if the appeal is allowable, or if the Examination Board revises their decision. You can also request a whole or partial refund if you agree to withdraw at critical points in the appeal procedure. 
  • You should think about what you want to get out of it, and how far you are willing to go in this slow procedure. There are two main steps in the procedure:
    • 1) an initial review by the Examination Board. They have 2 months to decide whether to revise their decision (either wholly regarding the passing/failing of the EQE or partially regarding the number of marks awarded for one or more papers). If they do not revise their decision, they send it to the Disciplinary Board of Appeal.
    • 2) a full consideration of the appeal by the Disciplinary Board of Appeal. In most Main Exam cases, you will not get a decision until after the next EQE. It is intended as a written procedure, but you can request oral proceedings.
  • Preston Richard has compiled an overview of the appeal procedure here. Be aware that the EPC does not automatically apply, so you need to make sure that everything is done in time. The rules for the EQE appeal are often inflexible, so if in doubt, check with the EQE helpdesk. For example:
    • Only 1 month to file the appeal AND all the grounds, and to also pay the fee. 
    • The 10-day rule (Rule 126(2) EPC) can be applied for the time limits
    • The fee cannot be paid using the EPO online software. It must be paid by bank transfer. It is also possible to pay the fee via the MyEQE portal, but you need to ask the EQE helpdesk to enable the option.
    • The appeal cannot be filed using the EPO online software and also not via the MyEQE portal. It either needs to be sent by post early enough that it arrives in time, or you can fax it to the EQE helpdesk. Be aware that the deadline for the appeal runs out for everyone on the same day, so you can have connection problems if you are trying to fax at the last moment. After faxing, you need to send an original immediately to the EQE secretariat.
  • Your exam has been marked by two markers independently and anonymously. 
    • In cases where the markers total score deviated too much from each other, it has either been re-evaluated by the same markers or marked by a third marker. This is not indicated in your decision, but I think these are marking sheets where the marks for each question are exactly the same for both markers.
    • For scores of 43-44 and 48-49, the Examination Board checks "fit-to-practice", to see whether you were unfairly marked down: 
      • they look for parts of your answer that did not directly score marks, but which indicate a level deserving of higher marks. These marks are indicated in the decision as 1-2 additional marks - i.e. not part of the exam marking sheets. About 20-40 candidates benefit each year per exam.
  • But mistakes do happen, and the markers may misunderstand what you have written. Study your exam answer in detail. Compare it to the model solution in the Examiners Report. Also look at the Candidate's Answer in your preferred language. Try to identify:
    • parts of your answer that do not appear to have been marked, or are identical to the Examiners Report but received far fewer marks than indicated
    • parts of your answer that are in the "wrong" place. For example, additional comments on claim 4 added after claim 6 arguments. 
    • incorrect labelling of answers or parts of answers
    • use of unusual words, or words in your own native language that may be unclear or ambiguous.
  • If a mistake like this has actually been made, the Examination Board will usually revise the decision within 2 months. A revised decision is again open to a new appeal.
  • If the case is more complicated, it will need to go to the Board of Appeal. It will then take several months before you receive their preliminary opinion, to which you can respond. The powers of the Disciplinary Board are limited - in most cases, they cannot do more than order the Examination Board to remark an answer, taking into account certain considerations. But such a remarking does not automatically lead to more marks. If you are about 3 to 5 marks short of your goal (pass or compensable fail), then you are in the range of typical marks increase where the Disciplinary Board agrees with you. 
  • Preston Richard has compiled an overview of recent appeal decisions here. There are several types of issue that you can address in your appeal:
    • 1) the Examination Board or Committees did not follow their own procedures. This is difficult to argue because you can only find fragments of the procedures in the case law.
    • 2) Unequal treatment of one or more candidates. For example, due to an error or a translation error in the En, Fr or Ge version of the exam. It does not matter which languages that you actually used because there is no way to determine who actually was affected by the translation error. So they award the compensation marks to all applicants who appealed that paper.
    • 3) Double penalisation. You should only be punished once for the same mistake. Unfortunately, parts of the exam will depend on another part (e.g. a claim may be dependent on another), so this principle can be difficult to apply in a way that avoids double penalisation in all cases. This is currently one of the stronger appeal arguments, particularly for B and C. 
    • 4)  Time loss due to technical issue. This is still developing based on appeals filed in 2021. In D 0037/21, reason 23, the Disciplinary Board suggested three ways to compensate an individual for a technical problem: a least generous approach, a more generous approach, and the average of the least and more generous. I have seen a couple of cases where the Examination Board has applied the least generous approach - the candidate's pc crashed in WISEflow, and it took some time to get the invigilator password and to get everything working again. That is based on an extrapolation of the actual number of marks scored:
      • compensation = (marks scored / (total exam time - time lost)) * time lost
      • For example, 20 minutes lost in B for a candidate with 42 marks may be compensated with (42 / (210 - 20)) * 20 => approx. 4 marks => 46 marks
    • Note that the examples in D 0037/21 are for a single-session exam. I do not know how these formulae will be applied to the split exams - D1-1, D1-2, D2 and C-1, C-2. They are still officially one exam with one score, but marks are awarded separately for each part. So it is possible to have a part-only calculation and complete paper calculation. 
  • I have copied the comments below from an earlier post, where a discussion started about appeal.

189 comments:

  1. Carol - 28/06/22 - 14:55July 11, 2022 at 11:23 AM

    This came today: "Dear candidate,
    Please be informed that the results of the EQE 2022 main examination will be available soon. We expect to provide them by 15 July 2022.
    All candidates will be informed by email once the results are available on myEQE."

    ReplyDelete
  2. Anonymous - 30/06/22 - 09:36July 11, 2022 at 11:24 AM

    Does anybody know what the problem is? I struggle to believe that it's IT-related; apparently, the results have been finalised, so setting the deadline of 15 July for simply sending the results to candidates seems a bit too much. Perhaps some marks need tweaking?

    ReplyDelete
    Replies
    1. Pete Pollard - 04/07/22 - 18:10July 11, 2022 at 11:27 AM

      I have heard a rumor that the delay is due to 900 complaints that were filed after the exam which need to be processed to determine the final number of marks.

      Speculating further on this: based on recent appeal decisions (like D 12/21 and D 37/21) from EQE 2021, the Examination Board should provide reasoning why individual complaints did not lead to extra marks, or why additional marks were awarded. For example, the board of appeal suggested to compensate for lost time due to technical issues (x marks per y minutes lost). This means that all the complaining candidates were in a position to file an allowable appeal based solely on the missing reasoning. They Examination Board would then have to provide the reasoning. Not all candidates would actually get extra marks, of course.
      Up to now, individual compensation marks have been added manually to the marking sheets without explanation - I can imagine that they need to bring structure into this and integrate it into the marking system.
      So, if this is true, some of the complaining candidates may benefit from this delay.

      Delete
    2. Anonymous - 04/07/22 - 18:27July 11, 2022 at 11:28 AM

      Shame, I really hoped they realised that the proposed mark scheme for Paper A was bonkers and decided to tweak it…

      Delete
    3. Pete Pollard - 04/07/22 - 18:32July 11, 2022 at 11:30 AM

      The complete marking scheme and all accepted variants are not published in the Examiners Report. They generally go with one general direction and the most common mark deductions.
      It has been tried before to publish more than one accepted solution, but it made it very difficult to read and created a lot of confusion.

      Delete
    4. Boris Johnson - 05/07/22 - 14:01July 11, 2022 at 11:31 AM

      I complained in 2021 and 2022 due to technical problems experienced: "fuzzy screen", wrong language paper, etc. I did not however appeal, since my law firm would not support me financially and I could not afford the fees myself. I feel I should have already passed B in 2021 so that this year should have been my last EQE. However, because I did not appeal, this year's B paper was a lot harder and so I will probably be stuck in the EQE hole again! How unfair!

      Delete
  3. Anonymous - 06/07/22 - 15:29July 11, 2022 at 11:32 AM

    results email arrived here at 14:38

    ReplyDelete
  4. Anonymous - 06/07/22 - 19:20July 11, 2022 at 11:33 AM

    Hi Pete, do you have an appeal template you can share please and any tips on what to include.

    I'm in a situation where I had IT issues and it was acknowledged and been awarded a couple of marks. I'm within 1 mark from passing and the issues affected me is between pass and fail. I feel that it had more of an impact and want to appeal.

    ReplyDelete
    Replies
    1. Anonymous - 06/07/22 - 19:24July 11, 2022 at 11:34 AM

      At the very least, I feel that it should be looked at by a 3rd marker as especially if it's just 1 mark off

      Delete
    2. Anonymous - 06/07/22 - 19:31July 11, 2022 at 11:35 AM

      The compensation seems very randomly allocated to me. IT issues affects people differently and in my case, it's literally between a pass and a fail. Is there anything I can do to convince them of this and award me 1 more mark. Or is it set in stone.

      Delete
  5. Pete Pollard - 06/07/22 - 20:24July 11, 2022 at 11:43 AM

    It is horrible to be so close. But think closely before filing an appeal - the procedure takes so long that you will already have taken the exam next year before you get the appeal decision. Most appeals are unsuccessful, so it can mentally be a negative path to follow which may affect your preparation for next year. For some people it is positive to at least complain, without having much expectation of success.
    You are aiming for extra marks due to the IT issues, due to mistakes in the exam, and/or based on your answer deserving more marks.
    I am glad to see that they have awarded some IT marks, but there should also be some reasoning given.
    If you were just 1 mark away, then your answer should have been reviewed to see whether you have been unfairly marked down. If the marks from the two markers were very different, then it should have already been reviewed by a 3rd marker to ensure consistency.

    You can also limit yourself to the first stage, which is asking the Examination Board to review their decision based on your appeal. This is most useful where there is clearly something wrong that has been missed during the marking, like part of your answer not being marked or not being correctly understood. The EB have to decide within 2m whether to revise the decision. (interlocutory revision) or to send it to the Board of Appeal.

    I don't think a template is useful because each appeal will be quite different. Try posting on the blogs and in the telegram groups to see if there are others who are also appealing the same exam, and do it together. If you are defending your answer as also correct, you will need to provide the same level of detail as you would need for an office action or an opposition.
    You can find some general information here (the fee has gone up): appeal against EQE
    And here are some appeal decisions: appeal decisions

    ReplyDelete
    Replies
    1. Anonymous - 06/07/22 - 20:37July 11, 2022 at 11:44 AM

      Thanks Pete. Any chance you can shoehorn appeals on 2 papers in one notice of appeal and save appeal fees that way.

      Delete
    2. Pete Pollard - 06/07/22 - 21:33July 11, 2022 at 11:45 AM

      You are appealing one decision, regarding passing or failing the whole EQE based upon the marks awarded for each exam. So, you can object to 1-4 exams in your appeal if they were taken this year.

      It is also possible to pay the fee via the MyEQE portal - you first need to as the EQE secretariat to enable the option.

      Delete
  6. Anonymous - 06/07/22 - 20:42July 11, 2022 at 11:47 AM

    My problem is that I couldn't see the claims for Paper C in each half for 5-10 mins at start plus the screen went blurry at some stage and crashed towards the end of part C1. As you know, it so crucial to see claims ASAP so that I meant I didn't finish properly some attacks.

    I don't know if they have looked at my paper carefully before or after awarding compensation. There is no indication in my letter at all that a 3rd person looked at it.

    I know it's an average of marks between 2 examiners but if I just take the highest marks awarded for each claim, it adds up to 44 before compensation applies. So another 2 marks awarded for IT issues may mean I could have 46 marks. I feel like at least a 3rd person should look at it when it's a difference between passing and failing based on a mark. Is it worth emailing the Exam sect first about this

    ReplyDelete
    Replies
    1. Anonymous - 06/07/22 - 20:43July 11, 2022 at 11:49 AM

      I should add its around 5-10 mins at start for both C1 and C2.

      Delete
    2. Jay - 06/07/22 - 21:01July 11, 2022 at 11:50 AM

      Correct me if I'm wrong Pete, but it sounds like this person had some credible IT issues, is extremely close anyway to compensable fail marks (45). Would the EPO have to look at the script more closely around 41-45 and 46-49, especially after the awarded marks for technical issues. I think there is a case of at least asking whether your script has been checked again. The Examiner often mark scripts (especially when there are hundreds to go through) in a hurry and can easily miss 1 or 2 marks here and there. I generally find that they can often find one or two marks additional marks.

      Delete
    3. Pete Pollard - 06/07/22 - 21:45July 11, 2022 at 11:51 AM

      I don't know what order they follow for the compensations, or how they can interact.
      Looking for 1-3 marks extra has a reasonable chance - it is much harder to get 10 marks.
      In some cases, it looks like it made a small difference if you were only 1-3 marks away from passing the whole EQE with high marks in the other exams. This is not a compensation, but it seems that some candidates in this situation were given benefit of the doubt when presenting substantive arguments regarding a single failed exam.

      Delete
    4. Pete Pollard - 06/07/22 - 21:49July 11, 2022 at 11:52 AM

      Also the compensation for IT issues has not yet been fully worked out yet, so if you had IT issues that were not your fault, it may be possible that they decide to award an extra 1 mark. But you should have already complained immediately after the exam, and preferably also during the exam through the widget.

      Delete
    5. Anonymous - 06/07/22 - 22:12July 11, 2022 at 11:53 AM

      I did immediately notify the EPO about the issues. I think is inadequate as the claims to paper C are so crucial and I couldn't see them for a while. Wiseflow crashing didn't help either.

      As for 44 marks, surely they should look at the paper on kts own and not consider other papers. If its enough to get 45 marks, it means there is no need to do that paper again next year which obviously has a huge impact on preparation for other papers etc... I did achieve high marks on 2 but need the other 2
      I don't think it's fair for the EPO to not look at your script at 44 marks because you have not pass the other 3. It still makes a difference as you could only need to do 1 other paper next year and not 2.

      Delete
    6. Pete Pollard - 06/07/22 - 22:34July 11, 2022 at 11:54 AM

      Sorry - my comments regarding other papers might be misleading. On their own, the scores on the other papers have absolutely no effect.
      But if you have a good case for getting more marks in an appealed paper anyway but it is still uncertain, it seems as if the Board of Appeal can use the scores in other papers as a kind of fit to practice benchmark to just tip the balance.

      Delete
  7. Anonymous - 06/07/22 - 22:14July 11, 2022 at 11:56 AM

    Is it possible to appeal 2 papers in one notice and pay 1 appeal fee?

    ReplyDelete
    Replies
    1. Pete Pollard - 06/07/22 - 22:17July 11, 2022 at 11:57 AM

      Yes - you are appealing one decision, regarding passing or failing the whole EQE based upon the marks awarded for each exam. So, you can object to 1-4 exams in your appeal if they were taken this year.

      It is also possible to pay the fee via the MyEQE portal - you first need to as the EQE secretariat to enable the option.

      Delete
  8. Anonymous - 06/07/22 - 22:18July 11, 2022 at 11:58 AM

    Hi Pete
    So if I have 2 papers left I got 44 marks on one. Is it worth appealing for that one to get it to 45 marks if possible or would EPO not consider it because you still have the other one left. I know people who got between 45 to 49 marks and don't do that paper again relying on compensible fail. I feel that the EPO should consider this situation some candidates find themselves in.

    ReplyDelete
    Replies
    1. Pete Pollard - 06/07/22 - 22:27July 11, 2022 at 11:58 AM

      You can appeal a fail and you can also appeal a compensable fail. Successfully appealing a compensable fail to get a pass puts you in a better position for the future when you do other papers.

      Delete
    2. Anonymous - 06/07/22 - 22:31July 11, 2022 at 12:00 PM

      So just to be clear, it's OK to appeal to get mark 44 changed to at least 45 marks if successful.

      Delete
    3. Anonymous - 06/07/22 - 22:33July 11, 2022 at 12:01 PM

      Would compensable fail apply if I successfully get 1 extra mark to 45 marks?

      Delete
    4. Pete Pollard - 06/07/22 - 22:42July 11, 2022 at 12:02 PM

      If you have 44 marks, normally when you appeal, you would normally request a pass (50 or more).
      If you want, and you would be happy with that, you can request just a compensable fail (45 - 49). There is more chance that the Examination Board will decide in interlocutory decision in your favor, but you still need to provide new argumentation that was not in your complaint.
      There are no guarantees - the Examination Board are not afraid to pass a request for 1 mark to the Board of Appeal.

      Delete
    5. Anonymous - 06/07/22 - 22:52July 11, 2022 at 12:03 PM

      Thanks Pete. Is it possible to do some sort of auxiliary request e.g. request pass 50 marks, request 45-49 marks? I will need to think carefully but I just want to know possible options at this stage.

      Delete
    6. Pete Pollard - 06/07/22 - 23:02July 11, 2022 at 12:04 PM

      Yes. It is common to indicate that the appeal should be withdrawn if the Examination Board does not revise their decision within 2m, because they do not want to go through the long procedure before the Board of Appeal.
      I don't know the exact rule, but it is possible to make withdrawal at any point (before sending to the Board of Appeal, before oral proceedings) dependent on a (partial) refund of the appeal fee.

      Don't forget there is a single deadline of 1 month for the notice of appeal, paying the fee AND filing the grounds.

      Delete
    7. Anonymous - 06/07/22 - 23:03July 11, 2022 at 12:04 PM

      As for new arguments. I'm not sure if the following will help and your thoughts would be much appreciated
      1) unreasonable compensation for IT issues. Not sure if this has been adequately considered given that there was a delay. Didn't tell me how it was calculated and why it was awarded such a mark.
      2) boundary marks - was the script checked again when compensation marks applied. I have no indication this was screen again by a 3rd examiner. Is that the correct procedure? I doubt it was checked again after award of compensation.
      3) script marked too harshly. Alternatives reasonably considered? This one I'm not sure about wlbut will check my own marks obviously to see where things could be different. There was a debate on CPA for 1 of the claim and I unfortunately started from one wrong.

      If anyone else has any other arguments for Paper C then will be much appreciated.

      Delete
    8. Pete Pollard - 06/07/22 - 23:17July 11, 2022 at 12:06 PM

      1) You should include this, and it will force them to provide something, but this is unlikely to get anything extra. The decisions, such as D 12/21 and D 37/21, discuss different possible compensation schemes which could be applied, so you should add your own suggestion.
      2) This can be useful, but you would be stronger if you have parts of your answer that did not appear to get marks. For example, an extra novelty or inventive step attack.
      3) Check everything exactly against the examiners report and their suggested answer. You can also look at the candidates solutions (although these will have some errors in them).

      Delete
    9. Anonymous - 06/07/22 - 23:41July 11, 2022 at 12:15 PM

      Thanks Pete. Will do
      One thing I have note is that for one of the claims, the solution state the optimal starting doc and a less desired starting doc. I opted for the less desirable one but only got 2 marks which seems pretty low. It's a partial problem and I did have ( I think good reason) why I think it was CPA. So I obviously followed the novelty analysis and given arguments on IS. The difference, the TE and OTP for first problem is what one would expect as it is also the missing feature in the first (optimal CPA doc) so I hoped to have more than 2 marks here. Would they consider or award reasonable marks for alternatives. The fact that they mention it in their solution as less desirable (to me, means it's still possible to start from it) so I think it should still obtain more than 2 marks.

      Delete
    10. JA - 07/07/22 - 00:17July 11, 2022 at 12:16 PM

      Hi Pete.
      Different person from Anon. I attacked claim 3 using an IS attack. They wanted added matter attack on claim 3. however, in my view, this is still a valid attack against claim 4 and should still be considered to knock out claim 3. In real opposition, this would be possible and alternative attacks foe the same claim is allowed. Is this a possible linenof argument that can be used or have been used before.

      Delete
    11. Anonymous - 07/07/22 - 00:21July 11, 2022 at 12:17 PM

      If I remember correctly A3a + A2 works extremely well against C3 too. Its also included in DP solutions. So C3 can be attacked using an alternative attack (not added matter).

      Delete
    12. Pete Pollard - 07/07/22 - 01:24July 11, 2022 at 12:18 PM

      To me, the huge loss of marks for the wrong CPA shows how artificial paper C has become. In a real-life opposition, that would be completely correctable. And in the oppositions I have been involved in, CPA is never discussed as a separate issue - the Board just asks you to present the inventive step attacks based on both of them. Once you see the steps, you can then decide which is the CPA. But if an inventive step attack is accepted from a not-so-close CPA, the claim still lacks inventive step. It is not necessary to reformulate it. It is supposed to be a test for "fit to practice".

      It is worth checking previous appeals for how "less-optimal attacks" are considered, But I have seen at least one appeal where detailed opposition-like argumentation was accepted, explaining how the candidate interpreted features in the prior art, but attacking a claim using an unexpected combination.

      Delete
    13. Anonymous - 07/07/22 - 06:49July 11, 2022 at 12:25 PM

      thanks Pete. if you or anyone know of this decision then that would be useful.

      Delete
  9. Anonymous - 06/07/22 - 23:05July 11, 2022 at 12:26 PM

    So if you keep it with the ED, you should technically know within 2 months of your appeal. I understand the delay is when it is referred to the BoA

    ReplyDelete
  10. Anonymous - 07/07/22 - 01:21July 11, 2022 at 12:27 PM

    If I follow the threads correctly above, one should request a straight pass in their appeal. A back up request is to compensable pass (45-49). Would that work?. If they don't think it's worth 50 but do think you should get 45, would this still be awarded.

    ReplyDelete
    Replies
    1. Pete Pollard - 07/07/22 - 01:29July 11, 2022 at 12:28 PM

      Yes - phrase it as a main request and an auxiliary request.

      Delete
  11. Papa A - 07/07/22 - 06:37July 11, 2022 at 12:29 PM

    Hi Pete - I'I think the solution to paper A this year is very super unfair in the marking and there are several issues which I plan to base my appeal around. Just wanted to see what you think. Its mainly around the independent method claim of making paper pulp which is worth 28 marks
    1) "Lignin" definition [008] "we only use raw plant material such as flax, hemp, straw, hay, thistles or nettles, or mixtures thereof, which does not contain lignin."
    One can derive from the above disclosure that wood contains lignin and the plant raw material does not contain lignin. Lignin: [008] is super ambiguous of whether it is inherent to raw plant material that it does not comprise lignin. Unfortunately, the word "which" in English is used to classify/specify and so I took the meaning that raw plant material is inherent that it doesn't contain lignin. If the wording was raw plant material THAT does not contain lignin, then I would've added this in. According to the solution, if you do not put in lignin-free, its a loss of 10 marks.

    2) [018] One of our inventors, Mr D. Séchard, first thought that these drawbacks could be tackled by adding the glue to the pulp directly in the vat of the stamper machine during beating. However, beating the pulp together with the glue resulted in a sharp increase in the viscosity of the mixture, hindering the circulation and homogeneous treatment of the pulp. The paper pulp obtained in this way was found to be unsuitable for our purposes. Normal stamper beating is said explicitly to be unsuitable, so surely you must refer to the features of the inventive stamper machine.
    Therefore, referring to the apparatus with angles other than 90 to obtain the technical effect in your method claim is a reasonable claim. As fit for practice candidates, one would expect to cover the inventive concept that achieves this technical effect in the method. A loss of 10 marks if you include the apparatus features that provide the inventive concept and based on the above that the text says it doesn't work across all stamper machines, I don't see how the EPO will let you get away with a general method claim without referring to the specific apparatus feature that allows the 90% circulation of pulp. I think this is dubious and arguable. it seems a loss of 10 marks is extremely harsh.

    Overall, I think 20 marks relating to method of making pulp can be arguable. Its interesting because DP solution also had raw plant material without specifying the need for it to be lignin-free and referred to the apparatus features in their method of making pulp.

    Apology for the long email but I wanted to get across my main reasonings.

    ReplyDelete
    Replies
    1. Anonymous - 07/07/22 - 06:44July 11, 2022 at 12:30 PM

      I don't see how limitation to the structural features that provides the technical effect of 90% pulp circulation can be seen as a major limitation when [0018] specifically mentions that traditional stamping machine do not provide this effect. The EPO has many times tried to limit claims to the specific structural feature that enables the effect. I [0019-0023], its clear that the 90% circulation is only achievable with the stamper machine of the present invention. No other stamper machine can do this so pointless to make it broad (not limit it to specific features) as the skilled person would find that the invention is not fully enabled across the breadth of the claim.

      Delete
    2. you can also use the translation error in 0023 german. in german there are three different independent advantages of the method, in english its one advantage causing two more advantages. this makes the adding of glue in english essential for the method, in german its optional because the other two advantages can be achieved with the increased circulation alone...

      Delete
    3. so the addition of glue is not described as essential in German?

      Delete
  12. Anonymous - 07/07/22 - 11:40July 11, 2022 at 12:31 PM

    If one withdraws at ED stage (i.e. after their decision in August), how much appeal fees do you get back?

    ReplyDelete
    Replies
    1. Pete Pollard - 07/07/22 - 13:38July 11, 2022 at 12:32 PM

      I have never seen a rule for this. EPC rules do not apply automatically to these proceedings, but they seem to apply something similar to Rule 103 EPC. You would have to ask the EQE secretariat how much would be refunded in that case.

      Delete
    2. You probably receive it all back. I filed an appeal last year, arguing that the B-exam markingsheet was inconsistent (which I still stand by). The Exam committee did not want to revise their markingsheet and let it go to the BoA for them to decide. However, since my appeal was on the contents of the exam, the BoA decided it was outside their scope and refused my appeal. I did not think about asking for a refund, and to my surprise I received a message from someone at the BoA 2 months ago, notifying me that I would probably receive all my money back if I was to send in a letter retracting my appeal. I did and indeed received a full refund! Thus in my mind it is definitely worth giving it a try, if you can spare the money for up to a year.

      Delete
    3. Thanks for that. I have seen many people get everything back, even quite late in the proceedings. In particular, they don't want to actually hold oral proceedings.

      Delete
  13. Anonymous - 07/07/22 - 22:39July 11, 2022 at 12:33 PM

    Can exam board still change or modify their mark schemes right now or is that closed off. Is it now set in stone? Even if they realise there is something that is inconsistent or an obvious error they haven't considered e.g. translation or ambitious wording that leads to misinterpretation.

    ReplyDelete
    Replies
    1. Pete Pollard - 07/07/22 - 23:26July 11, 2022 at 12:34 PM

      They are always open to correcting real and obvious mistakes in any document. They are also open to correcting obvious errors in an individuals marking.

      But the real process that guarantees complete consideration is an individual appeal. Even if they find an alternative interpretation, they will not update the examiners report. They will just consider how the interpretation affects that individuals' marking. There is also no mechanism for benefiting main exam candidates who do not appeal if it is decided that there are errors or ambiguities.

      Delete
  14. Axl - 08/07/22 - 00:58July 11, 2022 at 12:35 PM

    I have a question re compensable fails: I have obtained A - 58, B - 59, C - 43 and D - 46 marks, D is compesable fail.
    I think that my A and B will carry weight and provide marks to the compensable fail, even if my C exam next year is also a compensable fail. IPREE r(4)(a)-(c) says that I need 2 Pass (OK so far), no Fail (hopefully for C next year), and total of 200 (which, if I get at least 45 in C next year, will be also fulfilled). Is my interpretation correct? With my scores is it still necessary to sit D? The way i see it now is for honour reasons to get more than 46 marks, but maybe not necessary. Assuming the worst case scenario that I will fail C (less than 45 marks) next year, (and no new EQE format in 24), and that I pass C in 2024, my 46 marks in D will still be OK (available) to pass in 2024? I guess one question is is it OK to have 2 compesanble fails? And does a compensable fail remains as such for the next years? Thank you.

    ReplyDelete
    Replies
    1. Pete Pollard - 08/07/22 - 13:47July 11, 2022 at 12:37 PM

      Hi Axl, there is no reason to resit D. Even if you get the minimum possible on C (which is 45), you still have enough to pass everything. Yes - it is a minimum of 200 marks with max. two compensable fails.
      "Honor reasons" is not a good reason to risk resitting D - you will have problems motivating yourself to study, and your old score is overwritten, even if the new score is lower. There is also no shame in passing with 46 - the compensable fail system allows some balancing over exams because it is easy to screw something up on the day, even if you are well-prepared.
      Under the current system, you can keep D as compensable for as many years as you need to pass C.
      We do not yet know what the transitional arrangements for the new system will be, but they will be fair.

      Delete
    2. Axl - 08/07/22 - 18:50July 11, 2022 at 12:37 PM

      thank you very much Pete! I do not know why I had this idea that compensable fail vanishes the next year, despite reading a few times IPREE. anyways, focusing on C. Thanks again!

      Delete
    3. Pete Pollard - 08/07/22 - 19:18July 11, 2022 at 12:39 PM

      No problem. A lot of these provisions are unclear if you have not looked at them in detail. The EQE helpdesk will also answer such questions.
      ... and thanks for the Kopi Luwak :-)

      Delete
  15. Joris Bohnson - 08/07/22 - 21:10July 11, 2022 at 12:39 PM

    Passed at last! To those still short of the finish line: don’t worry! Keep calm, appeal if necessary, but carry on! Pete: you’re a legend!! Thanks for your help and advice along the way- absolutely indispensable! Seems for me timely given mr Johnson‘s departure from No 10, to be signing off now as well! Good luck everyone!! :-)

    ReplyDelete
    Replies
    1. Pete Pollard - 08/07/22 - 22:55July 11, 2022 at 12:40 PM

      A big congratulation! It looks like the passing rates (pass + compensable fail) was pretty high this year on all the exams.
      Thanks for the kind words - I am glad it helps. It is always very motivating when another bunch of recruits gets over the finish line. You can now get oon with the rest of your life :-).

      Delete
  16. Anonymous - 09/07/22 - 09:46July 11, 2022 at 12:41 PM

    Hi Pete.

    If you are appealing more than 2 papers in the same notice and let's just say the ED agrees with your findings on 1 of the papers but not the other/want to remit to BOA, does the whole thing gets remitted to BoA or is it just the paper in question. The other one that they agree with, they can pass a decision on.

    ReplyDelete
    Replies
    1. Pete Pollard - 09/07/22 - 13:15July 11, 2022 at 12:42 PM

      I don't know for sure. I think they send you a letter with the new decision asking you to confirm whether you still want the other papers to go to the board of appeal.

      Delete
    2. Anonymous - 09/07/22 - 15:08July 11, 2022 at 12:43 PM

      Thanks Pete. Do you have any contacts that would know for sure. I don't know if the whole decision e.g. for all papers will be delayed or just the one paper is delayed if it is referred to BoA and the other decision on Other papers by ED will stands. Off course if one paper is decided in your favour but the other is not, it seems odd to refer the whole thing to BOA

      Delete
  17. Anonymous - 09/07/22 - 10:25July 11, 2022 at 12:44 PM

    They need a much simpler software/requirement set up in 2023
    Wiseflow doesn't suit these papers especially paper C. It seems nobody takes much notice of candidates feedback.

    ReplyDelete
    Replies
    1. Anonymous - 09/07/22 - 10:26July 11, 2022 at 12:45 PM

      And they wonder why most trainees/recent Attorneys feel disengaged and dillusioned with EQE/EPO organisation.

      Delete
  18. Can't believe I blew it again... My law firm have said they are only willing to fund me a last time in 2023 and say the extra fees from 2024 onwards with the modular system make them rethink if they fund trainees fully in the future. They currently think to just find the D equivaltent modules. Anyone else had this (unfortnuate) response?

    ReplyDelete
    Replies
    1. What did you fail?
      Which fees are they no longer paying for? - to do the exams or training?
      Why do you think the modular system will be more expensive? The idea is that you will need fewer methodology courses, and you should be better prepared by your daily patent work.
      It is relatively common for a firm/employer to only pay for 1x legal training course

      Delete
  19. Thanks so much for this, it's really informative. I'm surprised by the "least generous approach" calculator - it assumes that the rate at which you gain marks isn't affected by the delay (i.e. that you'd have the same rate as if you had a clean run at the paper). Also it assumes that the rate you get marks in a paper is linear, which isn't the case for papers where you have to spend a lot of time reading and taking notes, then start writing your answers.

    I had substantial technical difficulties with paper C, which I think made it more or less impossible to pass:
    - I couldn't download and print the .pdfs of the papers, or my answers to the first half of the exam,
    - I got into the two parts of the exam ~14.5 and ~13 minutes late because I was in the widget with their IT support trying to fix the pdf issue (I also spent the whole of the break trying to fix it, so didn't get a break). With the 15 minute lock-out looming, I asked whether I should stay in the chat or just start the paper, and didn't get an answer (they replied that I shouldn't be using a VPN, which I wasn't).
    - Because of the additional stress caused by the delays, I then lost time during the exam worrying that I'd not be able to pass no matter what I did. It took me a long time to settle into the paper.
    - The chat then didn't work during the exam, so I couldn't ask the invigilator any questions/request bathroom breaks.
    - I had revised assuming I'd be able to hand-annotate and quickly flick through the paper/compare different sections, I then had to solely use the on-screen .pdf. This difference in styles slowed down working my way through the paper and made it harder to compare and contrast different sections.
    - Without the answers to the first part of the exam, I had to repeat the required calculations etc. for the notice of opposition in the second part, taking up valuable time.

    I duly filed my complaint and was awarded... 1 compensatory mark. This seems egregious, given the above. My answers had been marked at 34 marks, so this was nowhere near enough to get a compensory fail (I got 64, 52 and 55 in my other papers).

    Using the "least generous approach" for pure time loss, rounded to the nearest mark it should have been 3 marks compensation. Looking at D 0037/21, there's also what looks to be a "most generous approach" calculator (total marks/time of normal exam*time lost), which gives 8 marks. And that's just for pure time lost, let alone the other issues faced.

    As I'm 11 marks off a compensory fail, I suspect I'm too far off to appeal. What's your take on the situation, Pete?

    ReplyDelete
    Replies
    1. The BoA did comment tin D 0037/21 that the Examination Board can be more generous, and can compensate for stress. But I interpret the "least generous" to be the minimum that you can expect.
      But according to the Instructions To Candidates, you are theoretically responsible for the technical side, so I suspect they are placing some responsibility with you because of the "VPN" comment.
      This is all new, and they are slowly developing the case law. I don't think that they will apply the most generous approach for individual disruptions - the basic principle should still be an evaluation based on what you hand-in.
      Have you checked your answer against the Examiners Report to see if parts of your answer did not receive any marks, or received insufficient marks?

      Delete
    2. I had a similar problem with paper C. I couldn't load the paper for 5-10 mins each half and wiseflow then crashed at the end. They gave me 2 marks and so I'm 1 mark away from compensable fail. I feel like I've done everything necessary to make sure my IT equipment is as good as possible. Wiseflow crashing and pdf paper not loading is not my fault.im planning to appeal. The mark compensation in no way is sufficient for these issues

      Delete
    3. The awarding of 2 marks still means l am adversely affected by IT issues

      Delete
    4. If you don't agree with the number of marks awarded, either a fail or compensable fail, you can appeal. If they revise the decision to give you 1 more mark, you can appeal that as well if you do not think it is enough compensation.

      Delete
    5. Any indication that your script was checked by a 3rd Examiner after your compensation that led to 44 marks.

      Delete
    6. A 3rd examiner should check your script again when your are on the boundary of 45 or 50. This year results was delayed and rushed through so I doubt there was time to check it after omplaints were reviewed and compensation was applied

      Delete
    7. Thanks - added this step to the post above. For scores of 43-44 and 48-49, the Examination Board checks "fit-to-practice", to see whether you were unfairly marked down:
      they look for parts of your answer that did not directly score marks, but which indicate a level deserving of higher marks. These marks are indicated in the decision as 1-2 additional marks - i.e. not part of the exam marking sheets. About 20-40 candidates benefit each year per exam.
      You are correct - this means that the total score will depend on the order in which the compensations are applied.

      Delete
    8. Off course no one can be sure but the way the results were delayed, I would take a good guess that these marks were probably applied without Examiners knowledge and therefore, it appears that marks added which put you within the boundary have not been checked again by Examiners. I think that's a good ground for appeal as normal procedures have not been carried out and therefore, could be duly unfair to candidates this year compared compared other years

      Delete
    9. Is there a rule or guidelines that mentions examiners need to check when marks are on boundary?

      Delete
  20. There is no indication in the letter that paper C was checked by a 3rd examiner after time compensation was applied.

    ReplyDelete
  21. I also receive compensation for Paper C about IT issues. I think it was widespread relating to paper C. The letter didn't mention how they got to the decision or what calculations were used. Do they have to give you reasons as to which approach they used and why to calculate marks?

    ReplyDelete
    Replies
    1. In general, decisions have to be reasoned so that you can decide whether to appeal or not.
      Also, for the EQE, they are supposed to provide enough info to check your marking.

      Delete
    2. There was no Indication that my paper was checked again by a 3rd examiner (I scored 44). Would this be discussed. I have no idea how the calculated the compensation either. They should said it is based on time lost which could have been used to score marks. It's so vague and they did they take into account the stress caused. It happened to be in both C1 and C2.

      Delete
  22. Dear Pete, I passed the EQE this year and wanted to express my thanks to you for providing this blog. It has helped me a lot!

    ReplyDelete
    Replies
    1. A big congratulations! You can now stop having WISEflow nightmares and get on with your life ;-)
      Thanks for the kind words - I am glad it helped.
      Have a good summer :-)

      Delete
  23. A good question by Wieland in the telegram group:
    Is the time compensation formula based on the whole Part (a,b,c,d) or the specific subpart (C1,C2,d1-1,d1-2,d2)?

    ReplyDelete
    Replies
    1. That is a good question. The examples in D 0037/21 are only for a single session exam.
      D1-1, D1-2, D2 and C-1, C-2 are still officially one exam with one score, but marks are awarded separately for each part. So it is possible to have a part-only calculation and complete paper calculation.

      Delete
    2. It's so unclear how the ED applies D37/21. I lost some time during the break between C1 and C2 due to not being able to submit my C1 paper because wiseflow crashed. That time loss during the break should be taken into account in my view because I'm already being put at a disadvantage compared to other candidates by having a shortened break. Isn't this why they had to neutralise D1-1 last year because different candidates had different times and some had extended breaks whilst others didn't.

      Delete
  24. Alot of these issues stems from wiseflow. The EPO does not listen to candidates. They need to provide a much simpler system for candidates to use for these exams.

    ReplyDelete
  25. Does anyone know how long the ED will provide a decision on appealled decision. Someone told me it is around October? Is that correct

    ReplyDelete
    Replies
    1. After filing receipt of the appeal, the EB has 2 months to decide whether to revise or not.. So, yes - that will be at the end of September..

      Delete
  26. When is the last day to file an appeal?

    ReplyDelete
    Replies
    1. The date on your letter + 10 days + 1 month. Also take into account that it is the same time limit for a lot of candidates, so if you file at the last minute by fax, you will probably get a lot of delays.

      Delete
  27. In the Telegram EQE main group, there is a discussion about the novelty attack on claim 4 in the C paper. There could be a translation error in the German text..
    I also don't think it is legally weak if you use statements from the inventors about prior art as supporting "implicit disclosure".

    ReplyDelete
    Replies
    1. I don't have telegram
      Any chance someone can summarise or give a brief snapshot of claim 4 please. I also didn't see A6 as novelty destroying art for claim 4 so I lost 11 marks for it.

      Delete
    2. When reading the C ex.rep. i have another Question, how can a Patent be attacked by a document (A6) combined with "common knowledge" (implicit disclosure) known from the patent itself. There is no indication that this "common knowledge" was around prior to the patent. Secondly, the person s.i.t.a. has plenty of options to make an airtight bladder from the materials disclosed in the prior art for example Metals (a1,a2, a5, a6) or polyolefins (a6) or other elastomers (a5), there is no need for rubber.

      Delete
    3. I also struggle with the novelty attack to claim 4 with A6. Could any German native speaker could give opinions on the following:
      In the German version of A1 [0018], L23-26 reads
      " Wie bei allen Bällen, die aus Segmenten oder Teilstücken (7) zusammengenäht werden, wird eine z. B. aus vulkanisiertem Naturkautschuk hergestellte Gummiblase (6) bereitgestellt, die die Luftdichtheit gewährleistet."
      The Position of "zum Beispiel" makes the disclosure ambiguous.
      It can be understood as 1) for example, a rubber bladder made from vulcanized natural caoutchouc is provided to gurantee airtightness which means The rubber bladder is an alternative not a must 2) a rubber bladder made from for example vulcanized natural caoutchouc is provided to gurantee airtightness which means rubber bladder is necessary.
      The key for this novelty attack is this implicit disclosure of rubber bladder. Based on 1), the implicit disclosure can not be established.

      Delete
    4. For implicit disclosure has to be directly and unambiguously drivable.
      In order to justify the novelty attack, here needs to be interpreted as Must, not alternative.

      Delete
    5. If an alternative can be derived, this means rubber bladder is not a MUST to the ball. Then A6 does not disclose rubber bladder implicitly.

      Delete
    6. In German Gummiblase is only disclosed striclty connected to Natural coutchuk. But there are alternative Materials known to make something airtight. No other function is disclosed for the bladder.

      Delete
    7. It is fine to use the words of the patent to explain how the terms of the claim should be interpreted (like they do with the number of stainless steel wires).

      It is not allowed to combine the "opinion" of the inventors, which was not published before the patent was filed, with prior art. There is case law on this.
      Even with the English text, this is a losing attack - they refer in general to all (unspecified) balls that they are aware of. Even if they specified the disclosure, their own application cannot be prejudicial. I know it is an exam, but the answers should at least comply with the basic rules of patent law. This is teaching candidates something very wrong.

      You could use it and state that you would supply evidence that it is common general knowledge that they always have a rubber bladder. But technically that does not make sense anyway - there should always be some way of making them airtight, but that could also be a sealant applied to the stitches.

      The German version definitely suggests an alternative, so it cant be implicitly disclosed at all. This looks like a translation error.

      Was this why the C examiners report was not initially published with the rest?

      Delete
    8. CL I.C.2.7: In T 654/92 the board stated that the expression "background art" in the English version of R. 27(1)(b) EPC 1973 was to be interpreted as referring to prior art within the meaning of Art. 54(2) EPC 1973. The practice of starting out from art known to the applicant but not public at the claimed priority date was inconsistent with the requirements of the EPC. Any such art had to be ignored in an assessment of inventive step.
      (in this C exam, the inventors do not refer to a specified disclosure, so they are actually awarding marks for an inventive step attack using an A.54(3))

      In T 413/08 the board stated in the absence of any indication to the contrary, an acknowledgment of prior art by a patent proprietor might be accepted at face value. If a patent proprietor resiled (retreated - I had to look this up 😉) from an acknowledgment of prior art at a point in time which did not give rise to any procedural problems, the acknowledgement could no longer be relied upon.
      So, if in the procedure, the proprietor says - sorry, we did not mean "all" as in "every ball known in the universe" - then you cannot use it.

      Delete
    9. For C, you should always be aggressive and attack where you can. But you are always conscious of the time you might waste - so, a strong inventive step attack is, in general, better on the exam than a weak novelty one.

      Delete
    10. I don't understand how A6 used against claim 4 is implicitly disclosed in the EN version.

      Delete
    11. If it is found that there has been an error translation for A6 against C4? Would this apply to all candidates. I'm submitting an appeal against Paper C and wondering whether I should add this point into my appeal about translation errors between different versions and that all candidates should be treated equally. Does it matter that I only looked at EN version in the exam.

      Delete
    12. My understanding is that if the EPO finds that there is a translation error, and corrected this by a decision, they would apply their decision to at least all those who have appealled even if they didn't submit grounds of Appeal based on that specific point. Is that correct

      So if there is a translation error in DE compared to EN, that is a clear and obvious error and must be rectified. To be fair, it has to apply to all candidates (at least to those who have appealed).

      Delete
    13. To Pete. The "implicit" disclosure about rubber bladder was based on inventors recalling or providing a view of the prior art. This is exactly why I didn't use A6 for A54(3) against claim 4. It is not directly and unambiguously disclosed and you can't reply on the inventors views/opinions. Is there any case law for these sort of things.

      Delete
    14. There is no way for them to figure out who read which language, so everyone who appeals that paper is given extra marks, even if you did not mention it yourself. This is based on the principle that all candidates should be treated equally.
      Note that it is possible that this error was already taken into account in the marking. But in all previous cases, that was mentioned in the Examiners Report.

      Delete
    15. I'm not sure they have though. I got 14 out of 27 marks for claim 4. 16 marks for IS and 11 marks for Art 54(3). I got pretty much a carbon copy of my answer for IS to the model solution so assume 14 marks awarded for IS and didn't do Art 54(3) so got no marks for that.

      Delete
    16. With respect to using the inventors views and statements against themselves, see Case Law Book I.C.2.7 citations above.

      Delete
    17. From reading the case law above, I interprete that anything told by inventors should not be used as evidence of cgk (and cannot be used as implicit disclosure). Therefore, one cannot reasonably rely on A6 as Art 54(3) prior art. I don't think it matters whether you are reading if in DE or EN or FR. Plus, translation errors in DE version.

      Delete
    18. Sorry Pete
      I don't quite understand what you mean by awarding marks for inventive step using Art 54(3). How can that be possible? Is that correct from the Examiners?

      Delete
    19. Sorry - I jumped a few steps in my head. A6 does not disclose the rubber bladder, so it cannot be novelty destroying on its own, unless the skilled person can see that it is implicitly disclosed. As mentioned above, it is only implicitly disclosed if there are no alternatives. See Case law book II.E.1.3.3.

      So the statement in the application about the rubber bladder is closer to a teaching from "common general knowledge", because it is describing a common, well-known feature (rubber bladder) which would be highly likely used. Applying this teaching is basically an inventive step attack.. Don't worry about this inventive step explanation if it feels confusing :-)

      Delete
    20. One other thing to realise is that the implicit disclosure test (here used for A. 54) is essentially the same as used for A. 123(2) extension, and A. 87 (same invention). The EPO is very strict about what is considered added subject matter - A. 123(2) and they are very strict about same invention (A. 89). so they must be equally strict about what is disclosed in the prior art (A. 54)..
      See Case Law book: II.E.1.1: The Enlarged Board has stressed the importance of applying a uniform concept of disclosure (with reference to Art. 54, 87 and 123 EPC; see G 2/10, OJ 2012, 376, point 4.6 of the Reasons, citing G 1/03, OJ 2012, 436; see also G 1/15, OJ 2017, A82, with reference to G 2/98, OJ 2001, 413). See also e.g. T 330/14."

      And observations by the inventors are not automatically "common general knowledge". Inventors are not skilled persons, and they possess uncommon knowledge ;-)

      I think it is fine to award some marks for this attack, but not 11.

      Delete
    21. Following from anove, to summarise - the term "rubber bladder" is an example. You cannot say that this is implicitly disclosure because it is only described as an example and there are other alternatives. Is that correct?

      Delete
    22. In the German version, "... wird eine z. B. aus
      25 vulkanisiertem Naturkautschuk hergestellte Gummiblase (6)i", rubber bladder and vulcanized natural caoutchouc are all indicated as an example.
      You do this in a patent applcation to explicitly point out that other alternatives are possible - i,e, it should not be considered as abn essential feature.

      Delete
    23. I think it is super clear there is a translation error on this in the German version.

      Delete
    24. Yes. But just a mistake is not enough - it must have consequence that some candidates were disadvantaged.

      Delete
    25. I think iayone relying on the German text would be hesitant to attack novelty under A.54(3), and would therefore miss 11 marks.

      Delete
    26. It's clear that it does. If you read that text in German, you are not going to see A6 is A54(3) EPC as there is no implicit disclosure of rubber bladder as discussed above. That's 11 marks!!! A huge difference that can adversely affect candidates imo.

      Delete
    27. Even if one candidate misses this because of the translation error, that is sufficient of a disadvantage.

      Delete
    28. Yes. There is no way to determine who could have been affected, based on the language, so they usually award extra marks to everyone, or neutralize it in the marking.

      Delete
    29. Even without the translation error, this would be a weak attack in real-life. The statement cannot simply be interpreted as a "common general knowledge" that EVERY ball of this type in any prior art MUST have a BLADDER made of RUBBER. I don't know how they can award 11 marks for it.
      You can see how artificial the C exam is - they hide the comment in [0018] of description of A1: FIG. 2,. This clearly favors native speakers who can perhaps realise that such a statement could be the missing piece of the exam puzzle.. They can also remember it and scan quickly to find it again.
      Even the Candidates Answer in EN was unsure, and referred to the teaching of A3 about only being two types of ball.. Unfortunately, it is a novelty attack under Art. 54(3) - I wonder how many marks that candidate was awarded ?

      How can it be that this is what is used to determine whether candidates pass or fail?
      If they had wanted non-native speakers to find it and just use it in this way, they could have put it in the prior art section, of A1 and referred explicitly to the application A6. You are still testing the same attack and you will get the same argumentation - shouldn't we be testing that instead?

      Delete
    30. Completely agree with you Pete. These exams, especially A, B and C over the years are drifting far and far apart from real practice. It's a great pity that this is used as bar to let candidates enter the EQE register list. I only hope that the Examination Board rectify their mistakes in this one. I hope the modular system will be much clearer and fairer.

      Delete
    31. Pete. Do you think I should put the point around German translation error in my appeal letter?
      I would like them to consider the error even though I didn't read the German text in the exam.

      Delete
    32. I don't see why not. If its a clear and obvious error in the paper (EN, FR or DE) that adversely affect one or more candidates, why not point it out in your Appeal letter. It would be in breach of the rules

      Delete
    33. I don't think it matters which text you rely on in the exam. If there are errors between the papers that would disadvantage some candidates (even if not directly yourself), then it should be raised in the appeal for the ED to consider and rectify.

      Delete
    34. Yes - mention the translation error. You are provided with the exam in 3 languages. so that you can choose to read certain parts in a different language.
      See, for example, D 0008/21. The point to be made is that those using the German version would have been disadvantaged, and that should have been taken into account in the marking. The marking must treat all candidates equal unless there is a good reason for distinguishing (there is no reason here).
      By awarding 11 marks for an attack that a large group of candidates are less likely to get, the marking scheme is inherently flawed.
      The Examination Board must provide a marking scheme that does not disadvantage anyone who used the German text. As there is no way to determine objectively who used which language version and to what extent (the versions are provided to be used in case of doubt about the meaning of words or phrases), the marking scheme also cannot disadvantage anyone who made the same mistake.
      Equality also means that the marking scheme cannot take away marks already awarded for the "correct answer".

      Delete
    35. I would add that EQE 2021 paper D, the Examination Board neutralise D1-1 even though there was no effect for German candidates due to errors that candidates can only see D1-1 in German. So it must be fair and consistent for all candidates no where the translation error(and which language version) had occurred.

      Delete
    36. The "rubber bladder issue" seems to be based on the T0233/90. "Where the document according to Article 54(3) refers to "a usual manner" of preparing a product, it is permissible to use documents of reference such as handbooks, encyclopaedia or dictionaries in order to determine what the skilled person would have understood by such a reference on the effective date of the prior document". However, A6 did not mention "a usual manner" just a "traditional product". This product is not described. A1 is not a handbooks, encyclopaedia or dictionary. Also A1 only tells you that a rubber bladder is provided, it is not even described that the bladder is part of the final product and certainly not on the date of A6, where the "standard ball" did not even have a bladder (see A3). There is no definition of airtight or that all stichtings lead to imperfect airtightness. So the skilled person would take the standard glued bladderless ball and add a stiching.

      Delete
  28. For Paper A, there has been extensive discussion on DP blog about different interpretation as to whether lignin is implicit/inherent feature of the raw plant material n the EN version.

    ReplyDelete
    Replies
    1. Agree. I think the English version of the client letter (008) is not very clear. Raw plant material ... which does not contain lignin. The way it is written, it is an implied property of the raw plant material.

      Delete
    2. Yes, delta patent blog has extensive discussions on this. "Which" has a very different meaning in English.

      Delete
    3. Does anyone know of any case law relating to alternative interpretation due to ambiguous text or error translation? I think the ambiguity around the text in paragraph 008 in Paper A in English is unclear.

      Delete
    4. For example, D 0019/21 is based on B 2021, where the mark up of the amended claims was different between the languages. This results in unequal treatment: "this difference between the French and the English version may have had a significant impact on the answer paper of a candidate, and in this manner candidates writing the French and English versions had to write the Paper B 2021 under different conditions. The Board considers that these different conditions amount to an unequal treatment of the candidates. Such unequal treatment deserves some form of compensation, but such is not apparent from the Examiner's Report or the marking of the candidate. In this regard the Board refers to decision D 0008/21, points 10.1 to 10.3 and point 12.1 of the Reasons. The Board explicitly endorses these reasons of D 0008/21 and agrees with the conclusion stated in point 12.1 that the unequal treatment must be compensated."

      Delete
    5. at first I stumbled over "attacking a patent with the patent itself" in the ex.rep., and by now I found that this violates pretty much anything (GL and Case Law) about the "dates of disclosures, common general knowledge and implicit disclosures" then there is the problematic translation and thus an even stronger violation of the prerequisites for implicit disclosures in german

      Delete
    6. For Paper A, the text around lignin free in EN version is extremely unclear and seems to be a translation error or error. Unequal treatment of candidates.

      Delete
    7. Is the fact that the term "which" is used which in standard English meaning, it is intended to be used to state exactly what it is. So raw plant material which does not contain lignin is a confirmation statement. There is no need to put in "does not contain lignin" in the claim. This is unintended but it is still a translation error.

      Delete
  29. What is the DE or FR text like around lignin in paragraph 008. Is it clear that lignin is not inherent of the raw plant material?

    ReplyDelete
  30. [008] The first thing we would like to stress is that we do not use cotton and linen cloth rags to make our paper pulp. Using cotton and linen cloth rags requires bleaching, which is environmentally damaging. Of course, we do not use wood either. Instead, we only use raw plant material such as flax, hemp, straw, hay, thistles or nettles, or mixtures
    thereof, which does not contain lignin. We collect the raw plant material in our region. This makes our technology cheaper and more sustainable. We are currently striving to obtain an ecolabel.

    This is the English version of paragraph 008 in Paper A. From this, one can interpret that the absence of the lignin is an inherent property of the raw plant material itself. The use of the word "which" is unfortunate as it is read as a confirmation statement. Based on this statement, there is no need to add that the raw plant material which does not contain lignin. However, not doing so lost you 10 marks. Is it more clear in the DE and FR version?

    This could be a translation error but it is certainly unclear in the EN version

    ReplyDelete
  31. C 2022: Did anyone do the same attack on claim 4, using A6 and referring to A3 for "common general knowledge" instead of A1?
    Did you get all marks?

    ReplyDelete
    Replies
    1. I didn't, but would be interested to find out whether it is worth filing an appeal for not having the Cl4 A6 novelty attack. I had 11/27 for claim 4, with only the inventive step attack. If 11p would get neutralized due to incorrect novelty attack that is now marked as such, I would have passed (and I am a resitter just for C , after the 2020 hassle), so would be happy to know...

      Delete
    2. There are no guarantees, and you will most likely have to register for 2023 and take the exam before you get a decision.
      Just arguing that the novelty attack should not have been awarded 11 marks is usually not successful, and I don't know how black and white the case is that it is legally flawed. If there is one case where a statement from a patent was allowed to be used to successfully attack a claim (a million-to-one chance), then you are in the region of "very unlikely, but possible". It just does not match with the rest of the marking, where you lose a lot of marks for having the wrong closest prior art, and lose marks for "unexpected" inventive step attacks where you add something like the "implicit disclosure".
      Translation errors almost always lead to some degree of compensation, but there is no mechanism for anyone not appealing to benefit. Only those who appeal benefit from such a compensation. They do compensate everyone for Pre-Exam because it is not that much work to recalculate everything.

      The most important is mentally what it will do to your approach to the exam. Appeal is a long shot, and they don't make the process easy, so you can end up hating the exam. That will destroy your motivation for the future.
      Somebody mentioned that it is like having 2 chances to pass in 2023 - either directly or through appeal. You can also learn a lot from really studying your answer.

      Delete
    3. Don't the Examination Board reply to you within 2 months of appeal?

      Delete
    4. I did but not for claim 4 for claim 3. I got 16 points out of 27 for claim 4.

      Delete
  32. On my results letter there is no mention of compensation even though I had awful technical problems and complained. Should I expect a reply about this?

    ReplyDelete
    Replies
    1. If no compensation marks are indicated, then it is most likely that they reviewed the complaint and decided that no extra marks should be awarded. But they are supposed to provide the reasoning why the complaint was not successful.
      In general, they will only look at complaints and additional compensation if you scored less than 50 marks.

      Delete
    2. This comment has been removed by the author.

      Delete
    3. This is good to know! Altough I would have thought candidates still have a right to the compensation even beyond 50 marks.

      Delete
  33. Its criminal in my view that they deliberately hike up the appeal fees and drag out the whole appeal process. Its slow, inefficient and inflexible.

    ReplyDelete
  34. New topic for exam C: claim 1 is novel, because its distinguishable from a5.

    Thermal treatment is known at least since 26. Sept. 2018.
    Solvent treatment with residue to get rip of polyamid is known at least since 23.9.2018.

    However, the patent application A1 still states on the 25. Januar 2019 that any solvent leads to "better" void dimensions than other methods [0015, A1].

    therefore, the thermal treatment must
    -firstly always leave residue (otherwise the solvent method could not be better) and
    -secondly the residue must look kind of worse than with the solvent method (how else do you determine better control of the void dimensions).

    if we can't believe this statement in [0015], we can also not believe any other statement in A1, for example the one from [0018] about rubber bladders.

    ReplyDelete
    Replies
    1. I agree, I am planning some grounds for that. Also, the reference from GL clearly states that process step is relevant for novelty as long as these products structurally different and/or have different properties.
      Furthermore, the candidate's answer cites a case law which also states that this feature add novelty but not inventive step. Clearly, the citation from the candidate's answer is wrong.

      Delete
    2. more details: "firstly always leave residue (otherwise the solvent method could not be better)" this is because if the was no residue after thermal treatment, you can precisely control the dimensions by the width of the polyamid layer using thermal treatment, in this case the solvent method can't allow better control of the void dimensions.

      and if there's always residue, the better control of void dimensions should be read in the context of ([0016], A1, "For any hybrid yarn to be used in high mechanical stress situations it is important to have a void with well-defined dimensions which are constant along the yarn's length") meaning a solvent method leads to more homogeneous residue compared to thermal treatment

      Delete
    3. the solution to claim 2 in the exrep uses the statement from a1,0015 to combine a5 with a4, (remember 0015 is about all aggressive solvents) which means according to the exrep the person skilled in the art (short: psita) has a reason to go from thermal treatment to solvent treatment, because solvenmt treatment gives the psita better control over the void dimensions, which means solvent treatment gives you a better product in terms of void dimensions, which means claim 1 is new according to the exrep of claim 2.

      Delete
  35. I'm not worrying about EQE exams in 2023. I'm more worried about how the hell are we going to survive this winter. In the UK, it is expected to be £500 energy bill for a single month. Similar in Europe.

    ReplyDelete
    Replies
    1. Thick jumper, thermal undies!

      Delete
  36. I just realised i miss the due date for enrolling to EQE 2023, i'm so pissed off. Last year was 12!?? Results this year came out in July. There is nothing i can do about it?

    ReplyDelete
    Replies
    1. So sorry - as far as I know, they are very strict. You should call the EQE secretariat as soon as possible to check. The EQE regulations are not part of the EPC, so formally they do not need to accept late enrolment under any circumstances.
      But informally they will at least consider requests if there were provable situations beyond your control. And if you want them to take a decision that you can appeal, you will need to submit your requests in writing.

      Delete
    2. It seems as if many aspects of the EQE system are designed to be cold and cruel - I don't understand why they don't have a grace period (further processing with a 50% surcharge, for example. Like with patent deadlines.

      Delete
    3. Thanks a lot Pete, it was my
      Examination secretariat was pretty clear. The due date is follow stictly and no exception are allowed.
      But results this year came out very late, less than 2 months and due date for enrlo have been anticipate.

      I wrote to

      Delete
    4. I wrote to Ombuds Office of EPO, never heard about that Office before, I'm not sure if it could be helpful. Do you know something about it?

      Delete
    5. It is a recent appointment, so I have no experience. I have little experience with complaints to the EPO. It is certainly worth trying - let me know what happens.

      Delete
    6. I believe that a so earlier enrlolment was necessary when exam was taken on front, for venues, allocation of seats etc. But in a fully on-line examination they could give to candidates a little bit more flexibility.
      They should allow a late enrlolment upon the charge of an extra fee.

      Delete
    7. I fully agree - there should be some leniency for an exam that you can only take 1x per year with still 5 months before it is held. Particularly for resitters where all the details are already on file.
      Unfortunately, it is run like a non-profit or a civil service. If it was a business, they would be trying to maximise the number of enrolments, and they would be very happy to charge a penalty.
      Every law firm knows that client who has missed a deadline is desperate, and is very happy to pay both the office and agent surcharges. :-)

      Delete
    8. I sat two Paper this year but I believe to remember that it was not possible to to enrlol for EQE 2023 before the issue of the results. Anyone tried?

      Delete
    9. Anonymous September 9, 2022 at 2:07 PM. I was contacted by mail and Teams by Ombuds Office, they were very kind and asked fot all details and what would be my request. They contacted the Examination Board but they said me that a late enrol is only possible in case of "force majeure", not my case. Ombuds confirmed me that many other candidates are in this situation but the Board would not consider a late enrolment anyway.

      Delete
    10. It is good to know that there is some flexibility for force majeure, but it is a very unclear system. In cases like D 0015/09 where a resitter missed re-enrolment by a week, they emphasize that there is no legal basis for considering late enrolment at all. But then they do state that they could possibly consider requests based on force majeure: the cause "must pass the tests of externality (nothing to do with the appellant's conduct), unpredictability (if the event could be foreseen, the candidate/appellant is obliged to have prepared for it) and irresistibility (the consequences of the event must have been unpreventable). I guess the legal basis is D 0015/09 ;-)

      It is a very German approach to legal procedure - you are warned everywhere that you must be in time (like in the OJ 2022, A12 notice: "To ensure a smooth enrolment procedure for the EQE, candidates are reminded of the strict application of the closing dates for enrolment.
      Articles 121 and 122 EPC do not apply)" and "After 5 September 2022, it will not be possible to apply for
      enrolment for the main examination 2023", so they are covered. But most candidates are learning the EPC/PCT, and do not first spend a few hours studying how the EQE system deviates from the EPC.

      I still think there should be some flexibility (with a financial penalty) for those who are a few days late, especially as the exam is now online and there is a pre-registration system. You are also no longer allowed to have your employer or firm make the payment for you using the deposit account. Candidates are all busy working in this period, and the professional consequences of not being able to enrol can be very severe. And stuff just happens that don't meet the "force majeure" standard. How can a few days late prevent "timely and orderly preparation and organisation of the EQE" to be held in 5 months time?

      Delete
    11. what would be a case of force majeure?

      Delete
    12. I don't know if there are cases which have been allowed. These would not have been the subject of an appeal, so they are not published. I presume a medical emergency or natural disaster directly leading to not being able to enrol.

      The strict enrolment cut-off also assumes that enrolling and paying the fees is not a problem for candidates. But many candidates need permission from their employer to do the exam, or they have to wait to find out if their employer will pay the fees.

      Delete
    13. It is right to have a strict closing dates. Actually the enrlolling period for main EQE runs from April to September, but if, like this year, the Board takes 6 months to publish results (why? force majeure?), there is very little time left for those who have already sat the exam for the revision of the papers with Examiner's reports, decide whether or not to appeal and eventually sign up for the next EQE.

      Delete
    14. I heard from several people that it took longer than 2 months to find out whether their appeals were succesful or being remitted to the Board of Appeal. I can imagine that such candidates are waiting for these letters before deciding to enrol.
      Maybe that counts as "force majeure"?

      Delete
    15. I thought the two months deadline is quite strict. They should inform candidates really as many will make decisions based on this. Is this a substantial violation of procedure?

      Delete

    16. I tried to ask through the Ombuds Office of EPO for an extraordinary extension to the enrlolment precisely for the reasons that Pete listed and because not even two months of time was short for take action, but Nein! no chance.

      Delete
    17. Yes, it is strict - in the REE, it says ..."If the appeal is not allowed within two months from notification of the decision, it shall be remitted to the Disciplinary Board of Appeal of the EPO".
      But there is also no sanction defined in the REE, so it is not clear what you can do about it.

      Delete
  37. Any one heard from Appeal C? The two-month deadline expired...

    ReplyDelete
  38. The appeal deadline for the main examination of the EQE 2022 was 16 August 2022 (Article 24 REE).
    source - helpdesk

    ReplyDelete
    Replies
    1. However, the examination committee is supposed to give a feedback on the duly filed appeal by 06.09.2022, two months from the decision of the results.

      Delete
  39. I heard that lot of complaints was filed this year for the system is it true?

    ReplyDelete
    Replies
    1. I am not sure how it compares to last year. Probably most people complained about something because there is a very limited time after the exam to submit complaints. There is more discussion online about appeals, but I don't know if more appeals have been filed.

      Delete
    2. Is it possible a review of marks after the decision of appeals?

      Delete
    3. If you are not happy with the decision, you can appeal it. Some decisions will order part of your answer to be reviewed again, but that is no guarantee of more marks.

      Delete
    4. For everybody or just to for whom appealed?

      Delete
    5. Only for those who appealed. They make an exception for Pre-exam because they can easily recalculate the marks for all candidates, even if they did not appeal.

      Delete
  40. Hi Pete, do you teach or take a course or seminar of your own to preparation for the EQE?

    ReplyDelete
    Replies
    1. I am too busy with patent work to give as many courses as I used to. But it is great to be able to able to apply the knowledge to get client the patents they want :-).
      I will be doing the PCT part of the EPO's online course this year, and I give courses for formalities in English through Forum Institut. Hopefully, the EQELIBRIUM in-person courses will restart next year after COVID. I am also giving an in-hose course on D Methodology. And I also do paper correction for D.

      Delete
    2. Thank you for the info Pete!

      Delete
  41. As I wrote in other blog, I believe that Examination Secretariat, Examination Board etc. are not just fit to hold and run this examination. They struggle for run a very expensive examination once in a year for a few hundred of candidates and every year lot of further issues, complaints, appeals. This year pre-exam had 30 marks neutralized, last year paper D 25 marks. That is not fair.
    In my opinion EQE should be organized and run by a more competent and professional organization such as CEIPI or a University Institute with a supervision of IP professional commitee.

    ReplyDelete
    Replies
    1. Don't be misled by negative posts online - for the majority of candidates, everything went well. And this year was much better than last year. Behind the scenes, there are a lot of people doing amazing and admirable work, mostly in their free time - they are doing their best with the EQE system that we have. I have heard a lot of positive stories about issues being fixed informally before, during and after the exam, but it is a real patchwork of rules, appeal decisions and competing interests that they have to navigate. The role of the Secretariat and key members of the Board and Committees is also very underestimated - without their drive, there would be no consistency year-to-year, and there would have been no EQE at all in 2021 & 2022.

      What is not fair? They put the candidates first in these cases - they could have forced them to file appeals.

      Giving it to a separate organisation has been discussed before - it just creates other problems. Universities want to couple it to a paid course, but this makes it expensive and difficult to access for many EPC states (even if online). Having several universities do it gives consistency problems (look at the different study loads for the UPC litigation certificate).
      You also can't give it to a group of well-meaning volunteers - it is hard to find people who want to devote their free time or have employers who don't mind them using work time for their "hobbies". And it is a nightmare to run a project over several months, because volunteers sometimes have to choose daily work over EQE. And with new people all the time, there is little continuity.
      That is a weakness in the current system - they are reliant on the goodwill of the volunteers so criticism and expectations must be mild, and many of the volunteers feel that it is a thankless task. But on the other hand, it is a professional qualification in a multi-year training period. Every candidate has a technical degree, so they can pass with enough effort and time, and they should not have to worry about unexpected changes in exam levels, exam lengths and consistency, particularly for the same paper.
      I think the best option would be for the epi to be mainly responsible for the content and format of the exams, have a budget, have several paid full-time or part-time positions to make exams, and at least one leader with an EQE vision for the future ("EQE = epi exam") as part of an overall training & competence vision (+ formalities training + permanent education). The EPO can then be mainly responsible for what they do best - administration, organisation, planning and implemention.
      I don't know if this is an actual plan, but it seems to be the direction that they are heading towards.

      Delete
    2. Hi Pete,
      I dont want to blame anybody these are just my personal consideration by my personal experience sitting this EQE online, whitout being misled by anyone.
      I don't argue on the professionality and the effort of the people working for arrange the Exam and by all means EQE have to be focused and oriented to candidate, it would not be a smart thing for EPO forcing candidates to file appeals, it's like shooting on its foot.
      Moreover, EQE is not for free they are not giving us a favor, sitting all 4 paper cost 1000 + 400 for the past pre-exam, honesty candidates don't care if who working on arrange it spent his free time or as a hobby.
      If they had not been able to organize the 2021 2022 the EPO will report it to the EPC member states that will make their assessments.
      Probably a University wants to hold a course too but it's just what we all did, I think very few Patent Attorneys have passed EQE whitout CEIPI, DeltaPatens or similar training courses.
      You can understand then how very annoying could be when you struggle a couple of year for passing pre-exam after years of study, training, strict rules and burocracy see, year after, papers with 25 or 30 o marks neutralized randomly.
      Finally, my is a negative story, because the system issues making me loose lot of time during my paper and they did not resolve it during, nor before exame (I figure it out by myself how to continue), and they didn't give me a sinlge mark despite just the time spent chatting with the invigilator and at the telephone whit help.

      Delete
    3. I understand your frustration - there is no recognition of the individual impact of technical problems. Having to wait another year can screw up your career and salary growth, particularly if your fellow candidates have passed (that was my own experience - I failed B the first time and had to retake it. Passing ACD first time was considered nothing special :-(.
      The only financial figures I have seen are those for the old in-person exam in 2007 (CA/139/08):
      "All in all, the cost of the 2007 examination came to EUR 7 167 000.
      Of this, EUR 2 681 000 (estimate based on EPO cost levels) was contributed through the working time of professional representatives made available free of charge and EUR 108 000 (estimate based on EPO cost levels) through kind support from national patent offices in the form of premises and staff.
      Receipts from fees amounted to EUR 652 000.
      The remaining costs totalling EUR 4 378 000 were borne by the EPO."
      This was the main reason to introduce the Pre-exam - with the increasing number of resitters in the system, the EPO calculated that their costs would rise to 8 500 000 in 2018. The issue that has still not been resolved is that the EPC is silent over who will pay for EQE.
      I have no idea what the current online exams cost, but it must be a lot less for the EPO. However, the fees paid by candidates are still not enough to cover the costs, and they cannot find enough volunteers, so they are moving to exams that are less time-intensive to make and less time-intensive to mark.

      Delete
  42. It is understandable a little bit of frustration of fellow candidates who didn't pass papers and faced IT issues.
    I found a good progress this years spreading papers in 2 weeks, since we don't have a frontal exam anymore, there is no use to have all 4 papers in a row in 4 days.
    In my opinion new e-EQE papers should be taken twice in a year maybe splitting papers A-B or A-D in some date, C-D or B-C six month later.
    Unless you accept to schedule it in 2 year o more (plus a year for pre-exam), sitting all papers in the same time is a hard mental and physical stress.
    I think spread it during the year should be helpful for a better planning of the massive load of study you need.

    ReplyDelete
    Replies
    1. I fully agree with spreading exams over 2 weeks. The old "1 week" was mainly organisational when it was all held in-person. Having exams every 6 months would be a major advantage in spreading the load, and especially if you could resit. This would be possible with the new modular system if there are enough people available to make and check all these exams.

      Delete
  43. Hi Pete - do you have any tips to revise for paper A, anything to look out for/spot in papers. Apart from doing past papers, i'm not sure what else I could do. I didn't pick up the 6 independent claims they wanted last year, hence having to do it again this year.

    ReplyDelete